LSAT Explanation PT 41, S3, Q15: Midlevel managers at large corporations are

LSAT Question Stem

Each of the following, if true, supports the claim above EXCEPT: 

Logical Reasoning Question Type

This is a Strengthen question. 

Correct Answer

The correct answer to this question is E. 

LSAT Question Complete Explanation

First, let's summarize and analyze the argument in the passage. The argument states that midlevel managers at large corporations are unlikely to suggest reductions in staff in their own departments even when these departments are obviously overstaffed. This is the conclusion of the argument. The passage does not provide any premises to support this conclusion, which is what the question is asking us to do.

The question type is Strengthen Except, which means we need to find the answer choice that does NOT strengthen the argument. In other words, we're looking for an answer choice that either weakens the argument or is irrelevant to it.

Now, let's evaluate each answer choice:

a) The compensation paid to midlevel managers is greater when they supervise more workers.

This answer choice strengthens the argument by providing a reason (financial incentive) for midlevel managers to avoid suggesting staff reductions. So, this is not the correct answer.

b) Midlevel managers have less work to do when their departments are overstaffed.

This answer choice also strengthens the argument by providing another reason (less workload) for midlevel managers to avoid suggesting staff reductions. So, this is not the correct answer.

c) Staff morale and productivity often suffer when workers are laid off.

This answer choice strengthens the argument by pointing out the negative consequences (lower morale and productivity) of staff reductions, which could discourage midlevel managers from suggesting them. So, this is not the correct answer.

d) Departmental workloads at most large corporations increase and decrease significantly and unpredictably.

This answer choice strengthens the argument by suggesting that overstaffing might be necessary to handle unpredictable workloads, which could be another reason why midlevel managers don't suggest staff reductions. So, this is not the correct answer.

e) Many large corporations allow managers to offer early retirement as a means of reducing staff.

This is the correct answer choice because it does not strengthen the argument. The fact that early retirement is offered as a means of reducing staff has no direct impact on the midlevel managers' likelihood to suggest staff reductions in their own departments. This answer choice is irrelevant to the argument and does not strengthen it.

In conclusion, the correct answer is (e) because it does not strengthen the argument and is irrelevant to the conclusion that midlevel managers are unlikely to suggest staff reductions even when their departments are overstaffed.

Previous
Previous

LSAT Explanation PT 42, S2, Q26: Astronomer: I have asserted that our

Next
Next

LSAT Explanation PT 41, S1, Q18: Health officials now recommend that people